What is a laplacian of a laplacian?

  • Thread starter Thread starter tiredryan
  • Start date Start date
  • Tags Tags
    Laplacian
Click For Summary
The discussion centers on the mathematical concept of the Laplacian of a Laplacian, denoted as \(\nabla^2\nabla^2\). It questions whether this expression simplifies to \(\nabla^4\) or if it includes additional mixed derivative terms. The response clarifies that the second interpretation is correct, indicating that the Laplacian of a Laplacian does indeed involve additional terms from mixed derivatives. The order of differentiation does not affect the outcome, allowing for simplifications in the expression. Understanding these nuances is crucial for accurately applying the Laplacian operator in mathematical contexts.
tiredryan
Messages
50
Reaction score
0

Homework Statement


What is a laplacian of a laplacian?

Homework Equations


<br /> laplacian = \Delta = \nabla^2 = \frac{\partial^2}{\partial x^2}+\frac{\partial^2}{\partial y^2}+\frac{\partial^2}{\partial z^2}<br />

The Attempt at a Solution


Is the follow true?
<br /> \nabla^2\nabla^2=\nabla^4<br />
Also is this true?
<br /> \nabla^2\nabla^2=\frac{\partial^4}{\partial x^4}+\frac{\partial^4}{\partial y^4}+\frac{\partial^4}{\partial z^4}<br />
Or does it include additional terms?
<br /> \nabla^2\nabla^2=\frac{\partial^4}{\partial x^4}+\frac{\partial^4}{\partial y^4}+\frac{\partial^4}{\partial z^4}<br /> <br /> +\frac{\partial^2}{\partial x^2}\frac{\partial^2}{\partial y^2}+\frac{\partial^2}{\partial x^2}\frac{\partial^2}{\partial z^2}<br /> <br /> +\frac{\partial^2}{\partial y^2}\frac{\partial^2}{\partial x^2}+\frac{\partial^2}{\partial y^2}\frac{\partial^2}{\partial z^2}<br /> <br /> +\frac{\partial^2}{\partial z^2}\frac{\partial^2}{\partial x^2}+\frac{\partial^2}{\partial z^2}\frac{\partial^2}{\partial y^2}<br />

Thanks.
 
Last edited:
Physics news on Phys.org
Hi tiredryan! :smile:

The second one (and the order of eg ∂x∂y doesn't matter, so you can cut out three of them, and double the matching three). :wink:
 
Question: A clock's minute hand has length 4 and its hour hand has length 3. What is the distance between the tips at the moment when it is increasing most rapidly?(Putnam Exam Question) Answer: Making assumption that both the hands moves at constant angular velocities, the answer is ## \sqrt{7} .## But don't you think this assumption is somewhat doubtful and wrong?

Similar threads

Replies
2
Views
1K
  • · Replies 3 ·
Replies
3
Views
2K
  • · Replies 2 ·
Replies
2
Views
1K
  • · Replies 1 ·
Replies
1
Views
1K
  • · Replies 6 ·
Replies
6
Views
2K
  • · Replies 3 ·
Replies
3
Views
3K
Replies
6
Views
1K
Replies
2
Views
1K
  • · Replies 6 ·
Replies
6
Views
2K
Replies
2
Views
1K